$[0, 1]setminus mathbb Q$ can't be exhausted by Jordan sets












3












$begingroup$


We wish to show that $[0, 1]setminus mathbb Q$ can't be exhausted by Jordan sets. What I mean by this is that there's no nested sequence of Jordan sets $J_1 subseteq J_2 subseteq dots$ such that $bigcup_{k=1}^{infty}J_k = [0,1]setminusmathbb Q$



Jordan in this sense means Jordan Measurable (boundary is negligible)



Sadly, the fact that the boundary of the union is a subset of the union of the boundaries is only true for finite union, else this would be piece of cake.



How would you tackle this? It may be related to improper integration










share|cite|improve this question











$endgroup$

















    3












    $begingroup$


    We wish to show that $[0, 1]setminus mathbb Q$ can't be exhausted by Jordan sets. What I mean by this is that there's no nested sequence of Jordan sets $J_1 subseteq J_2 subseteq dots$ such that $bigcup_{k=1}^{infty}J_k = [0,1]setminusmathbb Q$



    Jordan in this sense means Jordan Measurable (boundary is negligible)



    Sadly, the fact that the boundary of the union is a subset of the union of the boundaries is only true for finite union, else this would be piece of cake.



    How would you tackle this? It may be related to improper integration










    share|cite|improve this question











    $endgroup$















      3












      3








      3





      $begingroup$


      We wish to show that $[0, 1]setminus mathbb Q$ can't be exhausted by Jordan sets. What I mean by this is that there's no nested sequence of Jordan sets $J_1 subseteq J_2 subseteq dots$ such that $bigcup_{k=1}^{infty}J_k = [0,1]setminusmathbb Q$



      Jordan in this sense means Jordan Measurable (boundary is negligible)



      Sadly, the fact that the boundary of the union is a subset of the union of the boundaries is only true for finite union, else this would be piece of cake.



      How would you tackle this? It may be related to improper integration










      share|cite|improve this question











      $endgroup$




      We wish to show that $[0, 1]setminus mathbb Q$ can't be exhausted by Jordan sets. What I mean by this is that there's no nested sequence of Jordan sets $J_1 subseteq J_2 subseteq dots$ such that $bigcup_{k=1}^{infty}J_k = [0,1]setminusmathbb Q$



      Jordan in this sense means Jordan Measurable (boundary is negligible)



      Sadly, the fact that the boundary of the union is a subset of the union of the boundaries is only true for finite union, else this would be piece of cake.



      How would you tackle this? It may be related to improper integration







      general-topology measure-theory improper-integrals






      share|cite|improve this question















      share|cite|improve this question













      share|cite|improve this question




      share|cite|improve this question








      edited Jan 1 at 23:49







      Oria Gruber

















      asked Jan 1 at 23:34









      Oria GruberOria Gruber

      6,53432460




      6,53432460






















          2 Answers
          2






          active

          oldest

          votes


















          1












          $begingroup$

          Better answer than the above one as I don't invoke measures.



          Notice that $X = [0,1]setminus mathbb Q$ doesn't have any interior points, $text{int}(X) = emptyset$, so if $J subset X$ then also $text{int}(J) = emptyset$.



          This means that $J subseteq partial J$. Now, if $J$ is Jordan measurable, then by definition $partial J$ is negligible, so $J$ is negligible.



          Thus if $X = bigcup_{k=1}^{infty}J_k$ then it follows that $X$ is negligible, since it's a countable union of negligible sets, and this is a contradiction, since $X$ is not negligible.






          share|cite|improve this answer









          $endgroup$





















            0












            $begingroup$

            Note that the inner Jordan measure of any subset of $A:=[0,1]setminusmathbb{Q}$ is $0$, since $A$ does not contain any positive length intervals. Hence, any Jordan subset of $A$ is of Jordan measure $0$, and hence of Lebesque measure $0$. Hence, any countable union of such sets is still of Lebesque measure $0$. Since $A$ has Lebesque measure $1$, this union can not equal $A$.






            share|cite|improve this answer









            $endgroup$













            • $begingroup$
              I would have prefered to solve this without invoking measures as we didn't get there yet. I don't know what you mean by inner measure
              $endgroup$
              – Oria Gruber
              Jan 2 at 9:09











            Your Answer





            StackExchange.ifUsing("editor", function () {
            return StackExchange.using("mathjaxEditing", function () {
            StackExchange.MarkdownEditor.creationCallbacks.add(function (editor, postfix) {
            StackExchange.mathjaxEditing.prepareWmdForMathJax(editor, postfix, [["$", "$"], ["\\(","\\)"]]);
            });
            });
            }, "mathjax-editing");

            StackExchange.ready(function() {
            var channelOptions = {
            tags: "".split(" "),
            id: "69"
            };
            initTagRenderer("".split(" "), "".split(" "), channelOptions);

            StackExchange.using("externalEditor", function() {
            // Have to fire editor after snippets, if snippets enabled
            if (StackExchange.settings.snippets.snippetsEnabled) {
            StackExchange.using("snippets", function() {
            createEditor();
            });
            }
            else {
            createEditor();
            }
            });

            function createEditor() {
            StackExchange.prepareEditor({
            heartbeatType: 'answer',
            autoActivateHeartbeat: false,
            convertImagesToLinks: true,
            noModals: true,
            showLowRepImageUploadWarning: true,
            reputationToPostImages: 10,
            bindNavPrevention: true,
            postfix: "",
            imageUploader: {
            brandingHtml: "Powered by u003ca class="icon-imgur-white" href="https://imgur.com/"u003eu003c/au003e",
            contentPolicyHtml: "User contributions licensed under u003ca href="https://creativecommons.org/licenses/by-sa/3.0/"u003ecc by-sa 3.0 with attribution requiredu003c/au003e u003ca href="https://stackoverflow.com/legal/content-policy"u003e(content policy)u003c/au003e",
            allowUrls: true
            },
            noCode: true, onDemand: true,
            discardSelector: ".discard-answer"
            ,immediatelyShowMarkdownHelp:true
            });


            }
            });














            draft saved

            draft discarded


















            StackExchange.ready(
            function () {
            StackExchange.openid.initPostLogin('.new-post-login', 'https%3a%2f%2fmath.stackexchange.com%2fquestions%2f3058990%2f0-1-setminus-mathbb-q-cant-be-exhausted-by-jordan-sets%23new-answer', 'question_page');
            }
            );

            Post as a guest















            Required, but never shown

























            2 Answers
            2






            active

            oldest

            votes








            2 Answers
            2






            active

            oldest

            votes









            active

            oldest

            votes






            active

            oldest

            votes









            1












            $begingroup$

            Better answer than the above one as I don't invoke measures.



            Notice that $X = [0,1]setminus mathbb Q$ doesn't have any interior points, $text{int}(X) = emptyset$, so if $J subset X$ then also $text{int}(J) = emptyset$.



            This means that $J subseteq partial J$. Now, if $J$ is Jordan measurable, then by definition $partial J$ is negligible, so $J$ is negligible.



            Thus if $X = bigcup_{k=1}^{infty}J_k$ then it follows that $X$ is negligible, since it's a countable union of negligible sets, and this is a contradiction, since $X$ is not negligible.






            share|cite|improve this answer









            $endgroup$


















              1












              $begingroup$

              Better answer than the above one as I don't invoke measures.



              Notice that $X = [0,1]setminus mathbb Q$ doesn't have any interior points, $text{int}(X) = emptyset$, so if $J subset X$ then also $text{int}(J) = emptyset$.



              This means that $J subseteq partial J$. Now, if $J$ is Jordan measurable, then by definition $partial J$ is negligible, so $J$ is negligible.



              Thus if $X = bigcup_{k=1}^{infty}J_k$ then it follows that $X$ is negligible, since it's a countable union of negligible sets, and this is a contradiction, since $X$ is not negligible.






              share|cite|improve this answer









              $endgroup$
















                1












                1








                1





                $begingroup$

                Better answer than the above one as I don't invoke measures.



                Notice that $X = [0,1]setminus mathbb Q$ doesn't have any interior points, $text{int}(X) = emptyset$, so if $J subset X$ then also $text{int}(J) = emptyset$.



                This means that $J subseteq partial J$. Now, if $J$ is Jordan measurable, then by definition $partial J$ is negligible, so $J$ is negligible.



                Thus if $X = bigcup_{k=1}^{infty}J_k$ then it follows that $X$ is negligible, since it's a countable union of negligible sets, and this is a contradiction, since $X$ is not negligible.






                share|cite|improve this answer









                $endgroup$



                Better answer than the above one as I don't invoke measures.



                Notice that $X = [0,1]setminus mathbb Q$ doesn't have any interior points, $text{int}(X) = emptyset$, so if $J subset X$ then also $text{int}(J) = emptyset$.



                This means that $J subseteq partial J$. Now, if $J$ is Jordan measurable, then by definition $partial J$ is negligible, so $J$ is negligible.



                Thus if $X = bigcup_{k=1}^{infty}J_k$ then it follows that $X$ is negligible, since it's a countable union of negligible sets, and this is a contradiction, since $X$ is not negligible.







                share|cite|improve this answer












                share|cite|improve this answer



                share|cite|improve this answer










                answered Jan 2 at 17:17









                Oria GruberOria Gruber

                6,53432460




                6,53432460























                    0












                    $begingroup$

                    Note that the inner Jordan measure of any subset of $A:=[0,1]setminusmathbb{Q}$ is $0$, since $A$ does not contain any positive length intervals. Hence, any Jordan subset of $A$ is of Jordan measure $0$, and hence of Lebesque measure $0$. Hence, any countable union of such sets is still of Lebesque measure $0$. Since $A$ has Lebesque measure $1$, this union can not equal $A$.






                    share|cite|improve this answer









                    $endgroup$













                    • $begingroup$
                      I would have prefered to solve this without invoking measures as we didn't get there yet. I don't know what you mean by inner measure
                      $endgroup$
                      – Oria Gruber
                      Jan 2 at 9:09
















                    0












                    $begingroup$

                    Note that the inner Jordan measure of any subset of $A:=[0,1]setminusmathbb{Q}$ is $0$, since $A$ does not contain any positive length intervals. Hence, any Jordan subset of $A$ is of Jordan measure $0$, and hence of Lebesque measure $0$. Hence, any countable union of such sets is still of Lebesque measure $0$. Since $A$ has Lebesque measure $1$, this union can not equal $A$.






                    share|cite|improve this answer









                    $endgroup$













                    • $begingroup$
                      I would have prefered to solve this without invoking measures as we didn't get there yet. I don't know what you mean by inner measure
                      $endgroup$
                      – Oria Gruber
                      Jan 2 at 9:09














                    0












                    0








                    0





                    $begingroup$

                    Note that the inner Jordan measure of any subset of $A:=[0,1]setminusmathbb{Q}$ is $0$, since $A$ does not contain any positive length intervals. Hence, any Jordan subset of $A$ is of Jordan measure $0$, and hence of Lebesque measure $0$. Hence, any countable union of such sets is still of Lebesque measure $0$. Since $A$ has Lebesque measure $1$, this union can not equal $A$.






                    share|cite|improve this answer









                    $endgroup$



                    Note that the inner Jordan measure of any subset of $A:=[0,1]setminusmathbb{Q}$ is $0$, since $A$ does not contain any positive length intervals. Hence, any Jordan subset of $A$ is of Jordan measure $0$, and hence of Lebesque measure $0$. Hence, any countable union of such sets is still of Lebesque measure $0$. Since $A$ has Lebesque measure $1$, this union can not equal $A$.







                    share|cite|improve this answer












                    share|cite|improve this answer



                    share|cite|improve this answer










                    answered Jan 2 at 0:02









                    SmileyCraftSmileyCraft

                    3,481516




                    3,481516












                    • $begingroup$
                      I would have prefered to solve this without invoking measures as we didn't get there yet. I don't know what you mean by inner measure
                      $endgroup$
                      – Oria Gruber
                      Jan 2 at 9:09


















                    • $begingroup$
                      I would have prefered to solve this without invoking measures as we didn't get there yet. I don't know what you mean by inner measure
                      $endgroup$
                      – Oria Gruber
                      Jan 2 at 9:09
















                    $begingroup$
                    I would have prefered to solve this without invoking measures as we didn't get there yet. I don't know what you mean by inner measure
                    $endgroup$
                    – Oria Gruber
                    Jan 2 at 9:09




                    $begingroup$
                    I would have prefered to solve this without invoking measures as we didn't get there yet. I don't know what you mean by inner measure
                    $endgroup$
                    – Oria Gruber
                    Jan 2 at 9:09


















                    draft saved

                    draft discarded




















































                    Thanks for contributing an answer to Mathematics Stack Exchange!


                    • Please be sure to answer the question. Provide details and share your research!

                    But avoid



                    • Asking for help, clarification, or responding to other answers.

                    • Making statements based on opinion; back them up with references or personal experience.


                    Use MathJax to format equations. MathJax reference.


                    To learn more, see our tips on writing great answers.




                    draft saved


                    draft discarded














                    StackExchange.ready(
                    function () {
                    StackExchange.openid.initPostLogin('.new-post-login', 'https%3a%2f%2fmath.stackexchange.com%2fquestions%2f3058990%2f0-1-setminus-mathbb-q-cant-be-exhausted-by-jordan-sets%23new-answer', 'question_page');
                    }
                    );

                    Post as a guest















                    Required, but never shown





















































                    Required, but never shown














                    Required, but never shown












                    Required, but never shown







                    Required, but never shown

































                    Required, but never shown














                    Required, but never shown












                    Required, but never shown







                    Required, but never shown







                    Popular posts from this blog

                    Human spaceflight

                    Can not write log (Is /dev/pts mounted?) - openpty in Ubuntu-on-Windows?

                    File:DeusFollowingSea.jpg